Generalization of Third Sylow Theorem

  • Thread starter Thread starter glacier302
  • Start date Start date
  • Tags Tags
    Theorem
Click For Summary
The discussion focuses on the application of the Third Sylow Theorem to a finite group G that is divisible by p^k. It examines how to demonstrate that the number of subgroups of G of order p^k containing a subgroup H of order p^j is congruent to 1 modulo p. Participants suggest leveraging the properties of p-groups and the containment of H within a Sylow p-subgroup. Additionally, there is a request for an example of a finite group with exactly p+1 Sylow p-subgroups. The conversation emphasizes the need to adapt the proof of Sylow's theorem for the specific case presented.
glacier302
Messages
34
Reaction score
0

Homework Statement



(a) Let G be a finite group that is divisible by by p^k, and suppose that H is a subgroup of G with order p^j, where j is less than or equal to k. Show that the number of subgroups of G of order p^k that contain H is congruent to 1 modulo p.

(b) Find an example of a finite group that has exactly p+1 Sylow p-subgroups.

Homework Equations



Theorem: Every p-group is contained in a Sylow p-subgroup.

Third Sylow Theorem: Let |G| = p^e*m where p does not divide m. Then the number of Sylow p-subgroups divides m and is congruent to 1 modulo p.

The Attempt at a Solution



I think that I should somehow be using the 3rd Sylow Theorem to prove (a). Also maybe the fact that since H is a p-group, it is contained in a Sylow p-subgroup, and any larger p-group containing H is also contained in a Sylow p-subgroup.

Any help would be much appreciated : )
 
Physics news on Phys.org
Rather than trying to apply Sylow's theorem, you should try to apply (with appropriate modifications) its proof.
 
Question: A clock's minute hand has length 4 and its hour hand has length 3. What is the distance between the tips at the moment when it is increasing most rapidly?(Putnam Exam Question) Answer: Making assumption that both the hands moves at constant angular velocities, the answer is ## \sqrt{7} .## But don't you think this assumption is somewhat doubtful and wrong?

Similar threads

  • · Replies 6 ·
Replies
6
Views
2K
  • · Replies 1 ·
Replies
1
Views
2K
Replies
2
Views
2K
Replies
5
Views
2K
  • · Replies 5 ·
Replies
5
Views
4K
  • · Replies 9 ·
Replies
9
Views
2K
Replies
1
Views
2K
  • · Replies 2 ·
Replies
2
Views
1K
  • · Replies 12 ·
Replies
12
Views
4K
  • · Replies 1 ·
Replies
1
Views
2K